Add last few weeks of Phys 102 solutions.
[course.git] / latex / problems / Serway_and_Jewett_8 / problem28.26.tex
1 \begin{problem*}{28.26}
2 The following equations describe an electric circuit:
3 \begin{align}
4   -I_1 (220\U{\Ohm}) + 5.80\U{V} - I_2 (370\U{\Ohm}) &= 0 \\
5   +I_2 (370\U{\Ohm}) + I_3 (150\U{\Ohm}) - 3.10\U{V} &= 0 \\
6   I_1 + I_3 - I_2 &= 0
7 \end{align}
8 \Part{a} Draw a [possible] diagram of the circuit.  \Part{b}
9 Calculate the unknowns sand identify the physical meaning of each
10 unknown.
11 \end{problem*}
12
13 \begin{solution}
14 \end{solution}